10
$\begingroup$

Last August I posted this on mathoverflow: https://mathoverflow.net/questions/71856/a-serendipitous-riemann-identity. I show the (slightly revised) equation below:

$$\zeta (3)=\frac{2\pi^4}{315} \prod _{n=1}^{\infty } \left(\frac{1}{(p_n){}^2-p_n}+1\right)$$

Since the constant, $\frac{2\pi^4}{315}$ contains $\pi$, which is known to be transcendental, wouldn't this prove that $\zeta(3)$ is transcendental?

I have calculated the product through the first million primes and Mathematica's Element[product,Rationals] returns True. Also, I built a continued fraction of 18,500,045 elements.

The product converges to http://oeis.org/A082695

A paper that uses the product: http://jtnb.cedram.org/cedram-bin/article/JTNB_2004__16_1_107_0.pdf

1 Answers 1

27

No, because an infinite product of rationals is not necessarily rational.

For instance, $$\prod_{n=1}^\infty \left(1-\frac{1}{4n^2}\right)=\frac{2}{\pi}$$

is not rational.

  • 0
    Wikipedia states the a number like $4\pi$ is transcendental. My product is made up of numbers like$\left\{\frac{3}{2}*\frac{7}{6}*\frac{21}{20}\right\}$. Would this do it?2012-04-20
  • 14
    No, it wouldn't. It doesn't matter where your numbers come from - an infinite product of rational numbers need not be rational. (It might be, but in general it won't. If it is then you need to prove it.)2012-04-20
  • 14
    What Bruno is telling you is that $(\pi)((3/4)(15/16)(35/36)\dots)=2$, from which you would like to conclude that 2 is transcendental.2012-04-20
  • 0
    @Bruno, as stated in OP, the product is rational.2012-04-20
  • 0
    @Rudy what makes you say that?2012-04-20
  • 0
    @Bruno: Mathematica's Element[f,Rationals]$=$ True2012-04-20
  • 1
    For the whole product or the partial products? Try it with my product and tell me what it gives you. In any case that's not an acceptable "proof"2012-04-21
  • 2
    @Bruno, you are right. At convergence, your product is not Rational and not Algebraic. At intermediate steps, it is both.2012-04-21
  • 1
    Dear @Rudy: Yes, the property of being rational (or algebraic, or transcendental, or irrational) does not pass over to the limit of a convergent sequence of elements having the property.2012-04-21